\documentclass[a5paper,10pt]{article} \usepackage{myXsim} \usepackage{tasks} % Title Page \title{DM2 \hfill \Var{Nom}} \tribe{TST} \date{\hfillÀ render pour le Mercredi 24 février} \xsimsetup{ solution/print = false } \begin{document} \maketitle \begin{exercise}[subtitle={Loi binomiale}] %- set p = round(random(), 2) Trois personnes s'apprêtent à passer le portique de sécurité. On suppose que pour chaque personne la probabilité que le portique sonne est égale à $\Var{p}$. Soit $X$ la variable aléatoire donnant le nombre de personnes faisant sonner le portique, parmi les 3 personnes de ce groupe. \begin{enumerate} \item Tracer l'arbre représentant le situation. \item Justifier que $X$ suit une loi binomiale dont on précisera les paramètres. \item Quelle est la probabilité qu'une seule personne fasse sonner le portique? \item Calculer puis interpréter les probabilités suivantes \[ P(X = 0) \qquad \qquad P(X \geq 2) \] \item Calculer l'espérance de $X$ et interpréter le résultat. \end{enumerate} \end{exercise} \begin{solution} %- set q = round(1-p, 2) \begin{enumerate} \item \begin{tikzpicture}[sloped] \node {.} child {node {$0$} child {node {$0$} child {node {$0$} edge from parent node[above] {\Var{q}} } child {node {$1$} edge from parent node[above] {\Var{p}} } edge from parent node[above] {\Var{q}} } child[missing] {} child {node {$1$} child {node {$0$} edge from parent node[above] {\Var{q}} } child {node {$1$} edge from parent node[above] {\Var{p}} } edge from parent node[above] {\Var{q}} } edge from parent node[above] {\Var{q}} } child[missing] {} child[missing] {} child[missing] {} child { node {$1$} child {node {$0$} child {node {$0$} edge from parent node[above] {\Var{q}} } child {node {$1$} edge from parent node[above] {\Var{p}} } edge from parent node[above] {\Var{q}} } child[missing] {} child {node {$1$} child {node {$0$} edge from parent node[above] {\Var{q}} } child {node {$1$} edge from parent node[above] {\Var{p}} } edge from parent node[above] {\Var{q}} } edge from parent node[above] {\Var{p}} } ; \end{tikzpicture} \item Chaque personne a 2 possibilités (1: fait sonner ou 2: ne fait pas sonner) et l'on fait passer 3 personnes ce qui correspond à une répétition identique et aléatoire. On peut donc modéliser la situation par une loi binomiale. \[ X \sim \mathcal{B}(3; \Var{0.76}) \] %- set p0 = round(q**3, 3) %- set p1 = round(3*p*q**2, 3) %- set p2 = round(3*p**2*q, 3) %- set p3 = round(p**3, 3) \item Probabilité qu'une seule personne fasse sonner le portique. On voit qu'il y a 3 branches qui correspondent à cette situation dont \[ P(X = 1) = 3 \times \Var{p}^1 \times \Var{q}^2 \approx \Var{p1} \] \item \[ P(X = 0) = \Var{q}^3 \approx \Var{p0} \] \[ P(X \geq 2) = P(X = 2) + P(X = 3) = 3 \times \Var{p}^2 \times \Var{q}^1 + \Var{p}^3 \approx \Var{round(p2 + p3, 3)} \] \item Il faut d'abord tracer le tableau résumant la loi de probabilité: \begin{center} \begin{tabular}{|c|*{4}{c|}} \hline Valeur & 0 & 1 & 2 & 3 \\ \hline Probabilité & $\Var{p0}$ & $\Var{p1}$ & $\Var{p2}$ &$\Var{p3}$ \\ \hline \end{tabular} \end{center} On peut alors calculer l'espérance %- set E = round(3*p, 3) \[ E[X] = 0 \times \Var{p0} + 1 \times \Var{p1} + 2 \times \Var{p2} + 3 \times \Var{p3} = \Var{E} \] On peut donc estimer qu'il y aura en moyenne $\Var{E}$ personnes qui feront sonner le portique sur les 3 personnes. \end{enumerate} \end{solution} \begin{exercise}[subtitle={Équation puissance}] Résoudre les équations et inéquations suivantes \begin{multicols}{2} \begin{enumerate} %- set a1 = randint(1, 50) \item $10^x = \Var{a1}$ %- set a2 = randint(1, 50) %- set b2 = randint(2, 20) \item $\Var{b2}^x = \Var{a2}$ %- set a3 = randint(2, 50) %- set b3 = round(random(), 2) \item $\Var{b3}^x \leq \Var{a3}$ %- set a4 = randint(2, 50) %- set b4 = round(random(), 2) %- set c4 = randint(2, 10) \item $\Var{c4} \times \Var{b4}^x = \Var{a4}$ \end{enumerate} \end{multicols} \end{exercise} \begin{solution} Les solutions ci-dessous ne sont pas justifiée car l'ordinateur ne sait pas faire. Par contre, vous vous devez savoir justifier vos réponses! \begin{enumerate} \item $x = \log(\Var{a1})$ \item $x = \frac{\log(\Var{a2})}{\log(\Var{b2})}$ \item Il faut faire attention quand on divise par un log car ce dernier peut être négatif ce qui est le cas ici. Il faut donc pense à changer le sens de l'inégalité. $x \geq \frac{\log(\Var{a3})}{\log(\Var{b3})}$ \item Il faut penser à faire la division à par $\Var{c4}$ avant d'utiliser le log car sinon, on ne peut pas utiliser la formule $\log(a^n) = n\times \log(a)$. $x = \frac{\log(\Var{round(a4/c4, 2)})}{\log(\Var{b4})}$ \end{enumerate} \end{solution} \begin{exercise}[subtitle={Étude de fonctions}] %- set a_ = randint(-10, 10) %- set a = a_*3 %- set x1 = randint(0, 50) %- set x2 = randint(-20, 20) %- set b = randint(-50, 50) %- set f = Polynom.from_coefficients([b, a*x1*x2, -a*(x1+x2)/2, a_]) Soit $f(x) = \Var{f}$ une fonction définie sur $\R$. \begin{enumerate} \item Calculer $f'(x)$ la dérivée de $f(x)$. \item Calculer $f'(\Var{x1})$ et $f'(\Var{x2})$. \item En déduire une forme factorisée de $f'(x)$. \item Étudier le signe de $f'(x)$ et en déduire les variations de $f(x)$. \item Est-ce que la fonction $f(x)$ admet un maximum ou un minimum? Si oui, calculer sa valeur. \end{enumerate} \end{exercise} \begin{solution} \begin{enumerate} %- set fp = f.differentiate() \item Dérivée de $f(x)$: $f'(x) = \Var{fp}$ \item \begin{align*} f'(\Var{x1}) &= \Var{fp(x1).explain() |join('\\\\&= ')} \end{align*} \begin{align*} f'(\Var{x2}) &= \Var{fp(x2).explain() |join('\\\\&= ')} \end{align*} Donc $x = \Var{x1}$ et $x=\Var{x2}$ sont des racines de $f'(x) = \Var{fp}$. \item On en déduit la forme factorisée suivante \[ f'(x) = \Var{a} (x - \Var{x1})(x-\Var{x2}) \] \item Pas de correction disponible \item À causes des branches extérieurs, la fonction $f(x)$ n'a pas de maximum ou de minimum. \end{enumerate} \end{solution} %\printsolutionstype{exercise} \end{document} %%% Local Variables: %%% mode: latex %%% TeX-master: "master" %%% End: